Tài liệu Bất đẳng thức Phạm Văn Thuận ppt

61 592 3
Tài liệu Bất đẳng thức Phạm Văn Thuận ppt

Đang tải... (xem toàn văn)

Tài liệu hạn chế xem trước, để xem đầy đủ mời bạn chọn Tải xuống

Thông tin tài liệu

Chương hu an Bất đẳng thức dạng bậc pv t Tính bậc (đồng bậc, nhất) tiêu chuẩn phải tính đến so sánh đại lượng Các bất đẳng thức cổ điển ta biết bất đẳng thức trung cỏc i lng trung bỡnh, Cauchy, Holder, Minkowski, ă Chebychev, , bất đẳng thức dạng đồng bậc Trong chương này, đề cập tới phương pháp để chứng minh bất đẳng thức đồng bậc, cách chuyển từ bất đẳng thức không đồng bậc bất đẳng thức đồng bậc Nắm vững vận dụng nhuần nhuyễn phương pháp này, chứng minh nhiều lớp bất đẳng thức sơ cấp 4.1 Bất đẳng thức dạng bậc Hàm số f ( x1 , x2 , , xn ) biến số thực x1 , x2 , , xn hàm bậc m với số thực t ta có f (tx , tx , , tx n ) = tm f ( x1 , x2 , , xn ), với t ∈ R − {0}, xi ∈ R, i = 1, 2, , n, m, n ∈ N, m = 0, n ≥ Số tự nhiên m gọi bậc đa thức đồng bậc Bất đẳng thức dạng f ( x1 , x2 , , xn ) ≥ 0, với f hàm gọi bất đẳng thức (bậc m) Khái niệm bất đẳng thức đồng bậc liên quan chặt chẽ với đa thức đồng bậc Thí dụ, hai đa thức sau hai đa thức đồng Đây chương sách Bất đẳng thức, Suy luận & Khám phá xuất tác giả Phạm Văn Thuận, Lê Vĩ 119 120 4.2 Đồng bậc hoá bất đẳng thức bậc đồng bậc g( x) = x5 + y5 + 8x2 y3 , f ( x) = x2 y + 4yx2 − 3x3 + 10y3 Từng đơn thức đa thức thứ có bậc năm, cịn đơn thức đa thức thứ hai có bậc ba Cũng cần ý đa thức kiểu f ( x) = ( x + 2y)3 + 101x2 khơng phải đồng bậc 4.2 Đồng bậc hố bất đẳng thức hu an Với bất đẳng thức có điều kiện, ta chuyển dạng bất đẳng thức đồng bậc Điều kiện cho trước thường hệ thức liên hệ biến số Từ giả thiết cho ta viết bất đẳng thức cần chứng minh dạng đồng bậc Bài toán 4.1 Cho số thực không âm a, b thoả mãn điều kiện a + b = 2, chứng minh dãy bất đẳng thức ≤ a + b2 ≤ a + b3 ≤ a + b4 pv t Chứng minh Ta chứng minh bất đẳng thức Mỗi vế bất đẳng thức bậc; mà ta thấy biểu thức điều kiện cho trước có dạng bậc Sử dụng giả thiết ta làm cân bậc bất đẳng thức Trước hết ta chứng minh ≤ a2 + b2 Thật vậy, nhân hai vế với hai, viết dạng tương đương ( a + b)2 ≤ 2( a2 + b2 ) Dễ dàng quy bất đẳng thức dạng ( a − b)2 ≥ Đối với bất đẳng thức thứ hai, ta viết lại dạng ( a2 + b2 )( a + b) ≤ 2( a3 + b3 ) Bất đẳng thức tương đương với a3 + b3 ≥ ab2 + a2 b, hay ( a − b)2 ( a + b) ≥ Điều hiển nhiên với a, b > Bất đẳng thức cuối làm tương tự Bài toán 4.2 Cho số thực a, b, c thoả mãn điều kiện a2/3 + b2/3 + c2/3 = 3, chứng minh bất đẳng thức a + b2 + c ≥ a / + b4 / + c / Hỏi dấu đẳng thức xảy nào? 121 4.3 Chuẩn hoá bất đẳng thức Chứng minh Để bỏ số mũ dạng hữu tỉ, ta đưa biến sau Đặt a1/3 = x, b1/3 = y, c1/3 = z Khi đó, ta cần chứng minh bất đẳng thức x6 + y6 + z6 ≥ x4 + y4 + z4 với điều kiện x2 + y2 + z2 = Sử dụng giả thiết ta viết bất đẳng thức cần chứng minh dạng tương đương 3( x6 + y6 + z6 ) ≥ ( x2 + y2 + z2 )( x4 + y4 + z4 ) Nhân khai triển nhóm số hạng cho ta ( x2 − y2 )2 ( x2 + y2 ) + ( y2 − z2 )2 ( y2 + z2 ) + ( z2 − x2 )2 ( z2 + x2 ) ≥ hu an Bất đẳng thức hiển nhiên 4.3 Chuẩn hoá bất đẳng thức Xét bất đẳng thức đồng bậc dạng f ( x1 , x2 , , xn ) ≥ g( x1 , x2 , , xn ), pv t f g hai đa thức đồng bậc Do tính chất hàm nhất, ta chuyển việc chứng minh bất đẳng thức việc chứng minh bất đẳng thức f ( x1 , x2 , , xn ) ≥ λ với x1 , x2 , , x n thỏa mãn điều kiện g( x1 , x2 , , x n ) = λ Chuẩn hóa cách thích hợp, ta làm đơn giản biểu thức bất đẳng thức cần chứng minh, tận dụng số tính chất đặc biệt số Nếu biết quan sát lựa chọn điều kiện thích hợp, nghĩa lúc có thêm giả thiết, ta có lời giải gọn gàng, sáng sủa Trong mục đưa số cách chọn điều kiện kiểu Bài toán 4.3 Chứng minh a, b, c > ( a + b − c)2 ( a + c − b) (c + b − a)2 + + c2 + (b + a)2 b + (c + a)2 a + (b + c)2 Chứng minh Bậc hai vế không Đặt x = a/( a + b + c), y = b/( a + b + c), z = c/( a + b + c), x + y + z = Do đó, ta viết bất đẳng thức cần chứng minh dạng (1 − 2y)2 (1 − 2x)2 (1 − 2z)2 + + z2 + (1 − z)2 y + (1 − y)2 x + (1 − x)2 (4.1) Chú ý t2 (1 − 2t)2 4t2 − 4t + 2(2t2 − 2t + 1) − 1 = = = 2− + (1 − t) 2t − 2t + 2t2 − 2t + 2t − 2t + 122 4.3 Chuẩn hoá bất đẳng thức Từ đó, ta viết lại bất đẳng thức (4.1) dạng 2x2 1 27 + + ≤ − 2x + 2y − 2y + 2z − 2z + Ta cần tìm số δ cho với < t < 1 ≤ +δ t− − 2t + 2t2 (4.2) sang vế trái quy đồng cho ta −18t2 + 18t − ≤ −δ t− 2t2 − 2t + hu an Chuyển Để ý t − nhân tử chung vế trái (4.2) nên ta viết lại bất đẳng thức dạng tương đương (4.3) t− 12 − 18t − δ ≤ 2t2 − 2t + Bây ta cần tìm δ cho biểu thức ngoặc thứ hai (4.3) nhận t − 12 −18t làm nhân tử Thế ta thay t = vào biểu thức δ = 2t2 −2t+1 thu δ = 54 Với giá trị δ, ta viết lại (4.3) dạng 12 − 18t 54 ≤ − − 2t + 2t pv t t− Bất đẳng thức tương đương với −(t − )2 (18t + ) ≤ Điều hiển nhiên 3 Vậy, ta có đánh giá sau 54 t− ≤ + 5 2t2 − 2t + Sử dụng ước lượng ba lần cho x, y, z ta có điều phải chứng minh Phép chứng minh hoàn tất Nói chung, bất đẳng thức có vế tổng ba phân thức khó khơng thể đánh giá phân thức Cách chọn cho phép ta làm điều khó khăn cách dễ dàng dựa tính chất bất đẳng thức, phân số, tam thức bậc hai Tổng quát hơn, ta chứng minh nhiều bất đẳng thức đối xứng đồng bậc cách đặt x = ka/( a + b + c) tương tự với y, z ta x + y + z = k, mà không làm tính đối xứng bất đẳng thức ban đầu Một cách tương 123 4.3 Chuẩn hoá bất đẳng thức tự, ta giải tốn sau thiết lập nhiều bất đẳng thức chứa biểu thức phân thức kiểu toán đề thi vô địch Hoa Kỳ năm 2003 (2b + c + a)2 (2c + a + b)2 (2a + b + c)2 + + 2 + (b + c)2 2a 2b + (c + a) 2c + ( a + b)2 8, với giả thiết a, b, c số dương Ta thấy tử thức mẫu thức phân thức vế trái đa thức bậc hai Vì vế trái bất đẳng thức có bậc khơng Khơng tổng quát, ta giả sử a + b + c = Và từ tiếp tục Ta xét thêm bất đẳng thức khác có dạng “tương tự" trên, chứng minh hu an (4b + c − a)2 (4c + a − b)2 (4a + b − c)2 + + 2a2 + (b + c)2 2b + (c + a)2 2c + ( a + b)2 Nhưng thật đáng tiếc kỹ thuật lại tác dụng với tốn Điều ta cô lập hay đưa phân thức dạng biến số thông qua điều kiện tổng ba số k Đây ý tưởng sở quan trọng cách làm Bài toán 4.4 Chứng minh a, b, c > ( a + b + c)2 + a + b2 + c 2 a + b3 + c a + b2 + c − abc ab + bc + ca pv t Chứng minh Bài tổng ba bất đẳng thức ngược chiều Biểu thức thứ ngoặc đạt giá trị nhỏ nhất; hai biểu thức đạt giá trị lớn Không giảm tổng quát ta giả sử ω = a2 + b2 + c2 , chọn ω = 3, sử dụng đẳng thức ( a + b + c)2 = a2 + b2 + c2 + 2( ab + bc + ca), a3 + b3 + c3 − 3abc = ( a + b + c)( a2 + b2 + c2 − ab − bc − ca) Do đó, ta viết vế trái bất đẳng thức dạng F= 2λ + + η( − λ ) − , 2λ λ = ab + bc + ca, η = 1/( ab) + 1/(bc) + 1/(ca), ta biết λ mặt khác sử dụng bất đẳng thức quen biết η 9/λ, F 2λ + + (3 − λ) − 2λ 2λ 2λ 12 + = −2 + λ 3, 124 4.3 Chuẩn hoá bất đẳng thức Do đó, để chứng minh F 4, ta cần λ /3 + 6/λ Thật vậy, sử dụng bất đẳng thức trung bình cộng trung bình nhân sau tách số hạng sau để có dấu đẳng thức, ta có λ + + λ λ λ 3 λ λ 1/3 = λ 1/3 Phép chứng minh hoàn tất Bằng cách đặt tương tự, ta thiết lập biểu thức đối xứng bậc chọn điều kiện đó, ước lượng giá trị biến số nhau, ta xây dựng bất đẳng thức hu an Bài toán 4.5 Chứng minh với số thực a, b, c 6( a + b + c)( a2 + b2 + c2 ) 27abc + 10( a2 + b2 + c2 )3/2 Chứng minh Rõ ràng hai vế bất đẳng thức có dạng bậc ba Nhưng tiếp tục lựa chọn thí dụ không hiệu biểu thức a2 + b2 + c2 có số mũ dạng hữu tỷ Nếu ba số khơng bất đẳng thức hiển nhiên Nếu có số khác khơng, ta đặt ω = a2 + b2 + c2 , giả sử | a| |b| |c|, chọn ω = để tránh dạng số vơ tỷ Thế bất đẳng thức có dạng 2( a + b + c) − abc 10 Sau thí dụ cho thấy phép chọn điều kiện thích hợp cho ta lời giải ngắn gọn, độc đáo pv t Bài toán 4.6 Giả sử a, b, c số thực dương, chứng minh bất đẳng thức ( a + b)(b + c)(c + a) ≥ ab + bc + ca Chứng minh Chọn ab + bc + ca = 3, ta cần chứng minh ( a + b)(b + c)(c + a) ≥ Nhưng ý ( a + b)(b + c)(c + a) = ( a + b + c)( ab + bc + ca) − abc Phép chứng minh hoàn tất ta 3( a + b + c) − abc ≥ Theo bất đẳng thức trung bình cộng trung bình nhân ta có − abc ≥ −1, ( a + b + c)2 ≥ 3( ab + bc + ca) Từ suy điều phải chứng minh Có hai câu hỏi cần đặt sau lời giải lại chọn số 3, chọn có lợi ích Cái tưởng chừng khó tốn có mặt hai thức khác vượt qua dễ dàng Bài toán 4.7 Cho số thực dương x, y, z đặt x + y + z = p, xy + yz + zx = q, xyz = r Giả sử p = 1, biểu diễn biểu thức x2 + y2 + z2 , theo p, q, biểu diễn x3 + y3 + z3 , x4 + y4 + z4 theo p, q, r 125 4.3 Chuẩn hoá bất đẳng thức Từ kết toán đây, người ta áp đặt điều kiện để xây dựng bất đẳng thức đối xứng ba biến có điều kiện Với toán bất đẳng thức đồng bậc ba biến, ta lựa chọn tuỳ ý (chỉ thôi) ba điều kiện p = 1, q = 1, r = Bài toán 4.8 Xét ba số thực không âm a, b, c thỏa mãn a + b + c = 1, chứng minh √ a a + b2 +√ b b + c2 +√ c c+ a2 ≤ L2 ≤ ( a + b + c) hu an Chứng minh Lời giải sau Võ Quốc Bá Cẩn Giả sử L vế trái bất đẳng thức Sử dụng bất đẳng thức Cauchy Schwarz, ta có a b c + + a + b2 b + c2 c + a2 = a b c + + a + b2 b + c2 c + a2 Do đó, ta cần chứng minh b c a + + ≤ a + b2 b + c2 c + a2 Bất đẳng thức lại viết dạng tương đương pv t b2 c2 a2 + + ≥ 2 b+a c+b a+c Viết bất đẳng thức dạng đồng bậc Lại áp dụng bất đẳng thức Cauchy Schwarz dạng Engel, cuối quy việc chứng minh a4 + b4 + c4 + 5( a2 b2 + b2 c2 + c2 a2 ) ≥ ( ab3 + bc3 + ca3 ) + 3abc( a + b + c) Theo bất đẳng thức trung bình cộng trung bình nhân a2 b2 + b2 c2 + c2 a2 ≥ abc( a + b + c) Và nên ta cần chứng minh a4 + b4 + c4 + 2( a2 b2 + b2 c2 + c2 a2 ) ≥ 3( ab3 + bc3 + ca3 ) Bất đẳng thức lại viết dạng ∑ ( a2 − c2 − 2ab + bc + ca)2 ≥ cyclic Phép chứng minh hoàn tất 4.4 Lớp hàm đối xứng sơ cấp ba biến 126 Phần làm bất đẳng thức đồng bậc trở nên không đồng bậc cách lựa chọn điều kiên thích hợp; có lợi cho biến đổi tính toán Bài toán 4.9 Cho ba số thực dương a, b, c cho a + b + c = 1, chứng minh bất đẳng thức 9( a3 + b3 + c3 ) − 10( a5 + b5 + c5 ) ≥ Bài toán 4.10 Chứng minh a, b, c > ta có bất đẳng thức ( a + b + c)2 + a + b2 + c 2 a + b2 + c a + b3 + c − ≤ ab + bc + ca abc hu an 4.4 Lớp hàm đối xứng sơ cấp ba biến Tất bất đẳng thức đối xứng ba biến số quy hàm đối xứng p = x + y + z, q = xy + yz + zx, r = xyz Trong tiết ta xét tốn bất đẳng thức, từ dễ đến khó, giải theo đường lối Sau viết bất đẳng thức cần chứng minh theo p, q, r, ta cần khảo sát bất đẳng thức theo ba biến p, q, r Điểm mạnh phương pháp xử lý bất đẳng thức đối xứng ba biến, chặt khó, ta không thực nhiều phép ước lượng trung gian thơ điều có nghĩa ta phải làm việc với nhiều bước tính tốn toán dạng phân thức, bậc cao Trước tiên, bạn đọc tự kiểm tra kết sau x2 + y2 + z2 = p2 − 2q, pv t x3 + y3 + z3 = p( p2 − 3q) + 3r, x4 + y4 + z4 = ( p2 − 2q)2 − 2(q2 − 2pr) Một vấn đề cần đặt thứ tự so sánh p, q, r Một nguyên tắc phải đảm bảo tính đồng bậc Với ba số a, b, c nghiệm phương trình bậc ba f ( x) = x + px2 + qx + r ta viết f ( x) = ( x − a)( x − b)( x − c) Khai triển đa thức ta ( x2 − xb − xa + ab)( x − c) = x3 − x2 ( a + b + c) + x( ab + bc + ca) − abc Bây sử dụng đồng thức ta  a + b + c = − p,  ab + bc + ca = q,   abc = −r Dựa vào đặc điểm đây, ta phát biểu nhiều tốn bất đẳng thức với giả thiết liên quan đến nghiệm phương trình bậc ba Trước hết, ta quan sát bất đẳng thức liên quan đến p, q, r Đó bất đẳng thức Schur 127 4.4 Lớp hàm đối xứng sơ cấp ba biến Định lý 4.1 (I Schur) Nếu x, y, z số thực dương t số thực, xt ( x − y)( x − z) + yt ( y − z)( y − x) + zt ( z − y)( z − x) ≥ Chứng minh Vì bất đẳng thức cần chứng minh có dạng đối xứng nên ta giả sử x ≥ y ≥ z Viết lại bất đẳng thức cho cách nhóm nhân tử chung, ta ( x − y){ xt ( x − z) − yt ( y − z)} + zt ( x − z)( y − z) ≥ Dễ thấy bất đẳng thức hu an Có số trường hợp riêng bất đẳng thức đáng ý trường hợp t = 1, 2, Lấy giá trị t khai triển đa thức ta thu trường hợp lý thú hữu ích Bài toán 4.11 Cho số thực dương x, y, z Đặt x + y + z = p, xy + yz + zx = q xyz = r, chứng minh bất đẳng thức p3 − 4pq + 9r ≥ 0, p4 − 5p2 q + 4q2 + 6pr ≥ 0, pq − 9r ≥ Chứng minh Ba bất đẳng thức viết lại dạng (4.4) (4.5) (4.6) x( x − y)( x − z) + y ( y − z)( y − x) + z( z − x)( z − y) ≥ 0, x ( x − y)( x − z) + y2 ( y − z)( y − x) + z2 ( z − x)( z − y) ≥ 0, x( y − z)2 + y( z − x )2 + z( x − y)2 ≥ pv t Dễ thấy hai bất đẳng thức ba đầu trường hợp t = 1, t = bất đẳng thức Schur Bất đẳng thức cuối hiển nhiên Bài toán 4.12 Cho số thực dương x, y, z Đặt x + y + z = p, xy + yz + zx = q xyz = r, chứng minh bất đẳng thức p2 ≥ 3q 2p3 + 9r ≥ 7pq p4 + 3q2 ≥ 4p2 q 2p3 + 9r2 ≥ 7pqr p3 ≥ 27r p2 q + 3pr ≥ 4q2 pq2 + 3q2 ≥ 4p2 q q3 + 9r2 ≥ 4pqr q2 ≥ 3pr p q ≥ 3pr + 4p2 q pq2 ≥ 2p2 r + 3qr p3 q + q3 ≥ 6pqr Từ kết ta phát biểu tốn dạng nghiệm phương trình bậc ba Chẳng hạn, ta xét toán sau Bài toán 4.13 Xét ba số thực a, b, c cho đa thức x3 + ax2 + bx + c có ba nghiệm Chứng minh 12ab + 27c ≤ 6a3 + 10( a2 − 2b)3/2 Hỏi dấu đẳng thức xảy nào? 4.4 Lớp hàm đối xứng sơ cấp ba biến 128 Bài toán 4.14 Chứng minh x, y, z > ( xy + yz + zx) 1 + + ≥ 2 ( x + y) ( y + z) ( x + z) Chứng minh Lời giải Hojoo Lee Sử dụng phép thay p, q, r Chú ý tính chất ( x + y)( y + z)( z + x) = ( x + y + z)( xy + yz + zx) − xyz = pq − r Ta chuyển bất đẳng thức cần chứng minh dạng sau theo p, q, r q ( p2 + q)2 − 4p( pq − r) ≥ ( pq − r) Biến đổi tương đương tính tốn cho ta hu an 4p4 q − 17p2 q2 + 4q3 + 34pqr − 9r2 ≥ pq( p3 − 4pqr + 9r) + q( p4 − 5p2 q + 4q2 + 6pr) + r( pq − 9r) ≥ Dễ thấy bất đẳng thức cuối theo toán Bài toán 4.15 Cho ba số thực dương x, y, z thỏa mãn điều kiện x + y + z = 1, chứng minh (1 − x2 )2 + (1 − y2 )2 + (1 − z2 )2 ≤ (1 + x)(1 + y)(1 + z) pv t Chứng minh Đây bất đẳng thức đối xứng, vai trị biến hồn tồn bình đẳng Theo cách đặt trên, ta có p = x + y + z, q = xy + yz + zx, r = xyz Thế bất đẳng thức cần chứng minh có dạng − 2( p2 − 2q) + ( p − 2q)2 − 2(q2 − 2pr) ≤ + p + q + r Chú ý p = nên bất đẳng thức lại có dạng − 2(1 − 2q) + (1 − 2q)2 − 2(q2 − 2r) ≤ + p + q + r, hay 2q2 − q + 3r ≤ Vì pq ≥ 9r nên q ≥ 9r Do đó, phép chứng minh hồn tất ta có 2q2 − q + q ≤ 0, điều tương đương với 2q2 − q ≤ Dễ thấy bất đẳng thức với ≤ q ≤ 1/3 Điều ta có p2 ≥ 3q Bài toán 4.16 Xét ba số thực dương x, y, z thỏa mãn điều kiện xyz = 1, chứng minh 1 + + + ≥ 2 (1 + x)(1 + y)(1 + z) (1 + x) (1 + y) (1 + z) 165 4.12 Một số kiểu ước lượng thông dụng Chứng minh Sử dụng đồng thức biết, ta quy bất đẳng thức dạng p( a − b)2 + q( a − c)(b − c) ≥ 0, p= ( a − c)(b − c) − − ( a + c)(b + c) ( a2 + b2 + c2 )( a + c)(b + c) a + b2 + c q= a + b + 2c ( a − b) − − ( a + c)(b + c) ( a + b2 + c2 )( a + c)(b + c) a + b2 + c Giả sử c = max( a, b, c) ta dễ có p, q ≥ Phép chứng minh hồn tất Bài toán 4.74 Chứng minh a, b, c > hu an a+b b+c c+a ( a + b + c)2 ≥ + + ab + bc + ca a+c b+a c+b Bài toán 4.75 Chứng minh a, b, c số thực dương a3 b3 c3 + + b c a ≥ a + b4 + c 4.12 Một số kiểu ước lượng thông dụng pv t Mục trình bày số kỹ thuật thiết lập đánh giá trung gian Một lần nữa, thơng tin dấu đẳng thức có tính chất gợi ý quan trọng định hướng tới ước lượng kiểu Bài toán 4.76 Chứng minh bất đẳng thức Nesbitt cho ba số thực dương Chứng minh pak a ≥ k , p = k = b+c a + bk + c k Thực vậy, thay p, k vào bất đẳng thức, ta cần chứng minh 2( a3/2 + b3/2 + c3/2 ) ≥ 3a1/2 (b + c) Lại theo bất đẳng thức AM-GM ta có a3/2 + b3/2 + b3/2 ≥ 3a1/2 b, a3/2 + c3/2 + c3/2 ≥ 3a1/2 c Cộng hai bất đẳng thức chiều, ta suy điều phải chứng minh Trở lại tốn, ta có điều cần chứng minh a3/2 b3 / c3/2 + 3/2 + 3/2 = a / + b3 / + c / a + b3 / + c / a + b3 / + c / 166 4.12 Một số kiểu ước lượng thơng dụng Bài tốn 4.77 Chứng minh a, b, c ≥ a3 a3 + + (b + c)3 b3 b3 + + (c + a)3 c3 c3 ≥ + ( a + b) Chứng minh Ta tìm p cho ap a3 ≥ p a + bp + cp a3 + (b + c)3 Bình phương hai vế bất đẳng thức, ta hu an a3 a2p + a3 (b p + c p )2 + 2a p a3 (b p + c p ) ≥ a3 a2p + a2p (b + c)3 , Đơn giản bất đẳng thức dạng (b p + c p )2 + 2a p (b p + c p ) ≥ a2p−3 (b + c)3 Ta dự đoán p = cho bất đẳng thức Thực vậy, Ta cần chứng minh b+c ≤ b2 +c2 t + a2 t2 − at3 ≥ 0, hay t2 ( t − a)2 ≥ 0, t = b + c Hoặc ta chọn b = c = 1, sau tìm p cho pv t a p − 2a2p−3 + ≥ Xét hàm f ( a) = a p − 2a2p−3 + Tính đạo hàm f ( a) cần f ′ (1) = Từ suy p = Cộng hai bất đẳng thức chiều nữa, ta thu bất đẳng thức cần chứng minh Phép chứng minh hoàn tất Bài toán 4.78 Chứng minh a, b, c, d số thực không âm p = a4 a ≥ 5 a2 + p(b + c + d)2 a4 + b4 + c4 + d4 Chứng minh Không tổng quát, ta cần chứng minh bất đẳng thức với 1 trường hợp a = 1, ta đặt x = b , y = c , z = d Bất đẳng thức cần chứng minh có dạng 1 ≥ + x5 + y5 + z5 + p( x4 + y4 + z4 )2 167 4.12 Một số kiểu ước lượng thơng dụng Bình phương hai vế, ta viết bất đẳng thức dạng 3( x10 + y10 + z10 ) + 6( x5 y5 + y5 z5 + x5 z5 ) + 6( x5 + y5 + z5 ) ≥ 5( x8 + y8 + z8 ) + 10( x4 y4 + y4 z4 + z4 x4 ) Sử dụng bất đẳng thức trung bình cộng trung bình nhân ta có 3x10 + 2x5 ≥ 5x8 , 6y5 z5 + 2y5 + 2z5 ≥ 10y5 z5 Làm bất đẳng thức tương tự, cộng chúng với ta suy điều phải chứng minh hu an Bài toán 4.79 Xét ba số thực không âm a, b, c cho a2 + b2 + c2 = 1, chứng minh √ a b c + + ≤ + bc + ca + ab Chứng minh Ta muốn tìm p cho √ p a 2a ≤ p + bc a + bp + cp pv t Với p = 1, ta cần chứng tỏ ( a + b + c)2 ≤ 2(1 + bc)2 , hệ trực tiếp (b + c − a)2 + 2b2 c2 ≥ Từ suy điều phải chứng minh Một câu hỏi tương tự tìm bất đẳng thức với số Về mặt kỹ thuật, ta thao tác hệt tốn trước Ta cần tìm p cho bất đẳng thức sau a a + (b + c + d)2 ≥ ap + bp ap + cp + dp Chỉ cần xét b = c = d = Thế bất đẳng thức có dạng √ a a2 + 15 ≥ ap ap + Bình phương hai vế bất đẳng thức rút gọn cho ta a2 p +2 + 6a p +2 + 9a2 ≥ a2 p +2 + 15a2 p Xét hàm f ( a) = 6a p +2 − 15a2 p + 9a2 Ta biết dấu đẳng thức xảy a = b = c = d = 1, ta tính đạo hàm f ( a) cho f ′ ( 1) = Giải phương trình cho ta p = 168 4.12 Một số kiểu ước lượng thơng dụng Bài tốn 4.80 Xét ba số thực khơng âm a, b, c, chứng minh a2 a2 + ab + b b2 + b2 + bc + c c2 + c2 + ca + a ( a + b + c) ≥ Chứng minh Lời giải Võ Quốc Bá Cẩn Với x ≥ 0, ta có x2 x2 Nếu x ≤ dạng + 4x + 1 ≥ x− bất đẳng thức hiển nhiên Nếu x ≥ , ta biến đổi hu an 36x4 ( x − 1)2 (15x − 4) − (3x − 1)2 = ≥ 4x2 + x + 4x2 + x + Sử dụng bất đẳng thức trên, thay x a/b, b/c, c/ a, ta có a2 b2 ≥ a − b, a2 + ab + b2 ≥ b − c, b2 + bc + c2 c2 ≥ c − a c2 + ca + a2 Cộng ba bất đẳng thức ta suy điều phải chứng minh Bài toán 4.81 Cho p = , chứng minh a, b, c > pv t a b c 1− p + b1 − p + c − p ) p + (b + c) p + (c + a) p ≥ p ( a ( a + b) Chứng minh Lời giải Võ Quốc Bá Cẩn Ta chứng minh với x > bất đẳng thức sau √ 4x3 √ ≥ 5x2 − 3 x +1 Thực vậy, x ≤ √ , bất đẳng thức tầm thường Nếu x ≥ f ( x) = Ta có 128x9 ( x3 + 1)(5x2 − 1)3 384x8 (1 − x)(2x2 − 3x − 3) ( x3 + 1)2 (5x2 − 1)4 √ + 33 ′ f ( x) = ⇔ x = 1, x= f ′ ( x) = √ , ta xét hàm 169 4.12 Một số kiểu ước lượng thông dụng Dễ dàng xác minh f ′ ( x) > với < x < √ < x < x > √ + 33 , √ + 33 , f ′ ( x) < với f ( x) ≥ ∀x ≥ √ f (1), lim f ( x) = x→∞ Do đó, bất đẳng thức Bây sử dụng bất đẳng thức cách thay số x với a b, b c, c a tương ứng, ta thu √ √ √ 2/3 2/3 b 2/3 2/3 c a32 √ ≥ a − b , √ ≥ b − c , √ ≥ c2/3 − a2/3 3 4 4 4 c+a a+b b+c hu an Cộng bất đẳng thức cho ta điều phải chứng minh Đẳng thức xảy với a = b = c Bài toán 4.82 Cho n ≥ số thực không âm a1 , a2 , , an thỏa mãn a1 + a2 + · · · + an = r, max{ } ≤ (1 − r2 )/2r n chứng minh n ∑ + a2 i =1 i ≥ n + r2 Chứng minh Với số thực dương x, ta tìm k cho pv t 1 ≥ + k( x − r) + x2 + r2 Bất đẳng thức tương đương với (4.40) (r − x ) r+x + k ≥ (1 + r2 )(1 + x2 ) Ta cần chọn k cho x = r nghiệm đa thức dấu ngoặc vuông, từ k = −2r/(1 + r2 )2 , ta viết (4.40) dạng tương đương (r − x)2 (2rx + r2 − 1) ≤ Điều x ≤ (1 − r2 )/2r Bây sử dụng bất đẳng thức vừa thiết lập cho số a1 , a2 , , an cộng bất đẳng thức thu được, ta có điều phải chứng minh Phép chứng minh hồn tất Bài tốn 4.83 Chứng minh với số dương a1 , a2 , , an thỏa a1 a2 · · · an = ta có bất đẳng thức a2 + + a2 + + · · · + a2 + ≤ n √ 2( a1 + a2 + · · · + an ) 170 4.12 Một số kiểu ước lượng thông dụng Chứng minh Xét hàm số f ( x) = √ có f ′ ( x) = x2 + − √ 2x + √ 2− √ ln x với x > Ta ( x − 1) −2x2 + x − − 2x2 2( x2 + 1) √ √ x 2( x2 + 1) 2x2 + x2 + f ′ ( x) = ⇔ x = Qua f ′ ( x) đổi dấu từ dương sang âm nên f ( x) ≤ f (1) = hay √ √ 2x − 2− √ ln x ∀ x > hu an x2 + ≤ ∀x > Sử dụng bất đẳng thức cho n số a1 , a2 , , an cộng lại, ta n ∑ i =1 a2 + ≤ i = √ √ ∑ − √ ∑ − √ n i =1 n i =1 2− √ 2− √ n ∑ ln i =1 ln( a1 a2 · · · an ) = √ n ∑ i =1 Vậy bất đẳng thức cần chứng minh Đẳng thức xảy a1 = a2 = · · · = an = pv t Cách làm cho ta ý tưởng giải lớp toán bất đẳng thức dạng hoán vị, tốn đối xứng có giả thiết tích số số Tuy nhiên, sau khảo sát số tốn khác, chúng tơi thấy kỹ thuật khơng hồn tồn vạn theo nghĩa có mà mặt hình thức tương tự trên, kỹ thuật lại đổ vỡ Lý hàm số cần xét khơng có tính chất ta mong muốn Bài tốn 4.84 Cho bốn số thực dương a, b, c, d thỏa mãn a2 + b2 + c2 + d2 = 1, chứng minh 1 1 + + + + 8( a + b + c + d) ≥ 24 a b c d Chứng minh Tìm k cho 1 + 8a ≥ + k a2 − a 171 4.12 Một số kiểu ước lượng thông dụng Chú ý đẳng thức xảy số Phân tích thành nhân tử, ta viết bất đẳng thức dạng (4.41) a− 8a − −k a+ a 2 ≥ Ta cần chọn k cho biểu thức ngoặc vuông nhận a = làm nghiệm, từ ta có k = Với k = 4, bất đẳng thức (4.41) tương đương với (1 − a)(2a − 1)2 ≥ Điều hiển nhiên với < a < Sử dụng liên tiếp bất đẳng thức bốn lần, ta có điều phải chứng minh Bài toán 4.85 Xét ba số thực dương x, y, z, chứng minh 16x + y+z 1+ 16y + z+x 16z ≥ x+y 1+ hu an 1+ Chứng minh Khơng tổng qt, ta chọn x + y + z = Ta chứng minh 16x ≥ + 2x 1+ 1−x Bình phương hai vế bất đẳng thức này, nhóm nhân tử chung, với lưu ý 1 x − nhân tử, ta thu bất đẳng thức tương đương x( x − )2 ≥ Làm hai bất đẳng thức tương tự, cộng chúng với ta có điều phải chứng minh Bài tốn 4.86 Xét bốn số thực khơng âm a, b, c, d, chứng minh a3 ≥ ( a + b)( a + c)( a + d) cyclic pv t ∑ Chứng minh Lời giải Naoki Sato Sử dụng bất đẳng thức trung bình cộng trung bình nhân, ta có ( a + b)( a + c)( a + d) ≤ Do a+b+a+c+a+d a3 ≥ ( a + b)( a + c)( a + d) = a+ a3 a+ b+c+d 3 b+c+d 3 Cũng có dạng này, bất đẳng thức sau chặt chưa giải theo cách Với bốn số thực dương a, b, c, d thỏa mãn a2 + b + c2 + d2 = 1, chứng minh 1 1 + + + + 10( a + b + c + d) ≥ 28 a b c d Sau lựa chọn k thích hợp, ta phân tích ( 2a − 1) ( − 3a) ≥ Điều max( a, b, c, d) ≤ Có lẽ phải biện luận trường hợp khác muốn theo hướng 172 4.12 Một số kiểu ước lượng thông dụng Không tổng quát, ta giả sử a + b + c + d = Thế a3 a+ b+c+d 3 a a + 4−a = 3a 2a + = = 27 a a+2 , đó, bất đẳng thức cần chứng minh trở thành ∑ cyclic a a+2 ≥ 27 Ta chứng tỏ x x+2 2x − 27 hu an ≥ với x ∈ [0, 4] Điều 2x − 2( x − 1)2 (− x2 + 6x + 4) = , 27 27( x + 2)3 √ √ − x2 + 6x + ≥ với − 13 ≤ x ≤ + 13, khoảng bao hàm đoạn ≤ x ≤ Do đó, 2( a + b + c + d) − 4 a = ∑ a+2 ≥ 27 27 cyclic − pv t x x+2 Phép chứng minh hoàn tất Bài toán 4.87 Xét n ≥ số thực dương thỏa mãn a1 + a2 + · · · + an = 1, chứng minh bất đẳng thức na2 − a1 + + na2 − a2 + +···+ na2 − an + n √ √ √ ≤ n − + √ ( a1 + a2 + · · · + an ) n Chứng minh Ta chứng minh x số thực cho ≤ x ≤ √ nx −x+1 ≤ 1−x+ x n 173 4.13 Ước lượng qua tam thức bậc hai Ta viết bất đẳng thức dạng 1− √ nx −x+1 ≥ x− x n Thực phép biến đổi liên hợp cho ta bất đẳng thức tương đương (1 + √ (nx − 1) x (nx − 1) x √ √ √ ≥ (1 + nx) nx nx2 − x + 1) nx2 − x + Dễ xác minh bất đẳng thức với ≤ x ≤ 1/n /n ≤ x ≤ Bây thay x a1 , a2 , , an cho ta n n √ ≤ n − + √ ∑ n i =1 na2 − + i hu an ∑ i =1 Phép chứng minh hoàn tất 4.13 Ước lượng qua tam thức bậc hai pv t Mục xét đến ước lượng bất đẳng thức thông qua tam thức bậc hai Việc xác định hệ số tam thực dựa hai thông tin quan trọng thông tin dấu đẳng thức yêu cầu phân tích, biến đổi tương đương Cách làm giúp ta giải toán bất đẳng thức tương đối chặt, khó Nhược điểm dễ thấy sau phép ước lượng trung gian, ta cần phải chứng minh bất đẳng thức có bậc cao Bài tốn 4.88 Cho ba số thực không âm a, b, c thỏa mãn a2 + b2 + c2 = 1, chứng minh 1 + + ≤ − ab − bc − ca Chứng minh Vì đẳng thức ta có max( ab, bc, ca) ≤ cho 1 +l x − + ≤ k x2 − 1−x Nhóm nhân tử chung cho ta bất đẳng thức tương đương x− 2 1 + l− ≤ k x+ 3 1−x Ta tìm k, l 174 4.13 Ước lượng qua tam thức bậc hai Ta cần tìm đẳng thức liên hệ k l cho g( x) = nghiệm 1 k x+ + l− 3 1−x Thay giá trị x = vào g( x) ta có l = − k Lại thay l vào g( x ) để g( x) phụ thuộc vào k Lúc ta cần chọn k cho nghiệm k− = 1−x Từ ta chọn k = , l = − Vậy ta có đánh giá sau hu an 1 ≤ x − − x− + 1−x Khai triển rút gọn cho ta 13 ≤ x2 − x + , với ≤ x ≤ 1−x 12 Sử dụng bất đẳng thức với x thay ab, bc, ca, cuối ta cần chứng minh 18( a2 b2 + b2 c2 + c2 a2 ) − ( ab + bc + ca) ≤ Viết bất đẳng thức dạng đồng bậc pv t 18( a2 b2 + b2 c2 + c2 a2 ) ≤ 5( a2 + b2 + c2 )2 + ( ab + bc + ca)( a2 + b2 + c2 ) Khai triển rút gọn cho ta ( a + b4 + c ) + ∑ cyclic a3 (b + c) + abc( a + b + c) ≥ 8( a2 b2 + b2 c2 + c2 a2 ) Chú ý ta có bất đẳng thức quen thuộc a4 + b4 + c4 + abc( a + b + c) ≥ 2( a2 b2 + b2 c2 + c2 a2 ) Phép chứng minh hoàn tất Bài tốn 4.89 Cho bốn số thực khơng âm a, b, c, d thỏa mãn a2 + b2 + c2 + d2 = 1, chứng minh 1 16 + + + ≤ − ab − bc − cd − da 175 4.13 Ước lượng qua tam thức bậc hai Chứng minh Từ giả thiết dễ dàng suy max( ab, bc, cd, da) ≤ Đẳng thức xảy bốn số nên ta cần chọn k, l cho 1 +l x − + , với ≤ x ≤ ≤ k x2 − 1−x 16 Biến đổi bất đẳng thức dạng x− 3 1 + l− ≥ k x+ 4 1−x Cần tìm đẳng thức liên hệ k l cho g( x) = 3 1 + l− k x+ 4 1−x hu an nhận làm nghiệm Thay giá trị x = vào g( x) cho ta l = 16 − k Lại thay l 4 vào g( x) g( x) cịn phụ thuộc k, ta viết bất đẳng thức dạng x− Chọn k cho 16 p − − x Rút gọn vế phải cho ta ≥ k− 16 1−x ≥ với x ≤ Từ ta chọn k = 32 Suy l = Vậy 32 x − + ≤ 1−x 16 pv t 32 10 1 ≤ x + với x ≤ 1−x 9 Bây sử dụng bất đẳng thức để chứng minh bất đẳng thức ban đầu Ta cần chứng tỏ 40 16 32 2 ( a b + b2 c + c d + d a ) + ≤ 9 Lưu ý a2 b2 + b2 c2 + c2 d2 + d2 a2 = ( a2 + c2 )(b2 + d ), theo bất đẳng thức trung bình cộng trung bình nhân, ta có điều phải chứng minh 10 Bài toán 4.90 Cho ba số thực không âm a, b, c thỏa mãn a + b + c = 1, chứng minh 10 Bạn 1 27 + + ≤ − ab − bc − bc Trần Lê Bách có đưa cách giải đơn giản việc sử dụng trực tiếp đánh giá sau 32 ≤ + − ab a2 b − 16 176 4.13 Ước lượng qua tam thức bậc hai Chứng minh Từ giả thiết tốn ta có max( ab, bc, ca) ≤ Đẳng thức xảy số Do đó, ta cần tìm k, l cho 1 +l x − + ≤ k x2 − 1−x 81 Ta tìm hệ thức liên hệ l = 81 − k, chọn l = 64 chuyển toán cần chứng minh dạng 57 64 , k= 27 16 Cuối 27( a2 b2 + b2 c2 + c2 a2 ) + 12( ab + bc + ca) ≤ hu an Đặt p = a + b + c, q = ab + bc + ca, r = abc bất đẳng thức chuyển dạng 27q2 − 54r + 12q ≤ Bây áp dụng bổ đề 4.35, ta có điều phải chứng minh Bài toán 4.91 Cho bốn số thực không âm a, b, c, d thỏa mãn a2 + b2 + c2 + d2 = 1, chứng minh 1 1 32 + + + ≤ − abc − bcd − cda − dab Chứng minh Từ giả thiết suy pv t max( abc, bcd, cda, dab) ≤ √ 3 Chú ý đẳng thức bất đẳng thức cần chứng minh thỏa mãn bốn số Coi x = abc, ta tìm k cho ước lượng sau thỏa mãn ≤ + k x2 − 1−x 64 Dễ dàng viết bất đẳng thức dạng tương đương (8x − 1) − (8x + 1) ≤ 7(1 − x) 49 Ta cần chọn k cho biểu thức ngoặc vuông nhận 8x − làm nhân tử Muốn vậy, k = 7(1− x648x+1) , với x = Từ suy k = 256 Với giá trị 49 )( k ta thu bất đẳng thức (4.42) 256 ≤ + 1−x 49 x2 − 64 177 4.13 Ước lượng qua tam thức bậc hai Thực bất đẳng thức ta viết dạng tương đương √ (8x − 1)2 (4x − 3) ≤ Điều với x ≤ 1/3 Sử dụng ước lượng (4.42) cho abc, bcd, cda, dab, ta cần chứng minh ( abc)2 + (bcd)2 + (cda)2 + (dab)2 ≤ 64 Đây bất đẳng thức ?? Phép chứng minh đơn giản cách nhóm sử dụng bất đẳng thức trung bình cộng trung bình nhân Bài tốn 4.92 Xét bốn số thực không âm a, b, c, d thỏa mãn a + b + c + d = 1, chứng minh hu an 1 1 1 96 + + + + + ≤ − ab − bc − cd − da − ac − bd 15 Chứng minh Từ giả thiết tốn, ta có max( ab, bc, cd, da) ≤ Đẳng thức bất đẳng thức cần chứng minh đạt số Từ đó, ta nghĩ tới lựa chọn k, l cho 1 16 +l x − + ≤ k x2 − 1−x 256 16 15 Nếu chuyển giả thiết toán dạng a + b + c + d = 4, ta cần tìm k, l cho 1 ≤ k( x2 − 1) + l ( x − 1) + 16 − x 15 225 − 2k, k = 2430 Ta có ước lượng pv t Từ đó, l = 1 123201 ≤ x + x+ 16 − x 2700 2430 131220 Bạn đọc tự hồn thành tốn này, coi tập Bạn đọc sử dụng kiến thức mục để giải bất đẳng thức ??, ?? sau Bài toán 4.93 Xét ba số thực dương x, y, z, tìm số p cho x+ x xp ≤ p x + yp + zp ( x + y)( x + z) Bài toán 4.94 Xét ba số thực khơng âm x, y, z, tìm số k cho bất đẳng thức sau x( y + z) kx + y + z ≤ + ( y + z)2 x+y+z x 178 4.13 Ước lượng qua tam thức bậc hai n+1 n−1 , Bài toán 4.95 Xét n ≥ số thực không âm a1 , a2 , , an , với r = chứng minh p = n+1 n p a1 a2 + r( a2 + a3 + · · · + an ) ≥ p p a1 p a1 + a2 + · · · + an Bài tốn 4.96 Xét tám số thực khơng âm x1 , x2 , , x8 thỏa mãn x3 + x3 + · · · + x = 1, chứng minh ≤ 64 − xi x j xk ≤i < j

Ngày đăng: 26/01/2014, 15:20

Từ khóa liên quan

Tài liệu cùng người dùng

Tài liệu liên quan